jessie uses 20 liters of gasoline to travel 200kilometers, how many liters of gasoline will he use on a trip of 700 kilometer?

Answers

Answer 1

Answer: He will use 70 liters of gasoline on a trip of 700 kilometer.

Step-by-step explanation:

Given : Jessie uses 20 liters of gasoline to travel 200 kilometers.

We assume that the rate of consumption of gasoline is contant.

By UNITARY METHOD, the amount of gasoline required for 1 kilometer distance = [tex]\dfrac{20}{200}=\dfrac{1}{10}[/tex] liter

Now, the amount of gasoline required for 700 kilometer distance = [tex]700\times\dfrac{1}{10}=70[/tex] liters

Hence, He will use 70 liters of gasoline on a trip of 700 kilometer.

Answer 2

To calculate how many liters of gasoline Jessie will use for a 700 km trip, the fuel efficiency is first determined to be 10 km/L from the given data. Using this information, it is found that Jessie will need 70 liters of gasoline for the 700 km trip.

Jessie uses 20 liters of gasoline to travel 200 kilometers. To determine how much gasoline Jessie will use on a trip of 700 kilometers, we first need to find the fuel efficiency of the vehicle. By dividing the distance traveled by the amount of gasoline used, we get the fuel consumption rate:

Fuel efficiency = Distance / Gasoline

Fuel efficiency = 200 km / 20 L

Fuel efficiency = 10 km/L

Now we know that the vehicle uses one liter of gasoline to travel 10 kilometers. To find out how many liters are needed for a 700 km trip, we apply the following:

Gasoline needed = Trip distance / Fuel efficiency

Gasoline needed = 700 km / (10 km/L)

Gasoline needed = 70 liters

Therefore, Jessie will use 70 liters of gasoline to travel 700 kilometers.

Learn more about Fuel Efficiency Calculation here:

https://brainly.com/question/28314501

#SPJ3


Related Questions

if your average stride length is 2.6 feet how many strides will it take you to walk to grandparents house

Answers

The distance from your house to your grandparents house is 3.67 miles or 19377.6 ft.

So the total number of strides needed would be:

strides needed = 19377.6 ft / (2.6 ft / stride)

strides needed = 7,452.92 = 7,453 strides

the posted speed limit in many parts of europe is 100km/hr. what is the equivalent speed limit in miles per hour?

Answers

1 m = 1.609 km 1 km = 0.621371 miles .62 miles x 100 = 62 miles per hour

List the next three terms in the following sequence: 1, 2, 4, 8, 16, 32,... a. 48, 80, 128 b. 64, 96, 160 c. 64, 128, 256 d. 46, 92, 184

Answers

C
because they are doubling


C because the numbers multiply by 2

Apply Mathematics (1)(
a. A company’s weekly revenue R is given by the formula R = - p^2+ 30p, where p is the price of the company’s product. The company is considering hiring a distributor, which will cost the company 4p + 25 per week.

Answers

The values of price p for the product to remain profitable must be less than 1 and 25.

The value of P that will maximize the profit is 13.

Maximum function

For the given product to remain profitable, the revenue must continue to be greater than the cost that is R(p) > C(p)

Given the following parameters:

[tex]R(p) = - p^2+ 30p\\C(p)=4p+25[/tex]

If R(p) > C(p), then;

[tex]- p^2+ 30p>4p+25\\p^2-26p+25<0[/tex]

Factorize to get the value of p

[tex]p^2-p-25p+25<0\\p(p-1)-25(p-1)<0\\(p-1)(p-25)<0\\p<1 \ and \ p<25[/tex]

Hence the values of price p for the product to remain profitable must be less than 1 and 25

To maxmimize the profit, dP/dp = 0

Profit = Revenue - Cost

Profit = [tex]- p^2+ 30p - 4p - 25[/tex]

Profit = [tex]-p^2+26p-25[/tex]

Differentiate the function and equate to zero;

[tex]dP/dp = -2p+26\\-2p + 26 = 0\\-2p = -26\\p=13[/tex]

Hence the value of P that will maximize the profit is 13

Learn more on cost and revenue functions here: https://brainly.com/question/25638609

Mr. Rose spent $63 for a sport jacket and a pair of slacks. If the jacket cost $33 more than the slacks, how much did he pay for each?
Which system of equations represents the word problem if j is the jacket price and s is the price of the slacks?

j + s = 63 and s - j = 33
j + s = 63 and j - s = 33
js = 63 andj/s = 33

Answers

the equation would be j+s=total
we know that the total is 63, and the jacket cost s+33
it is addition and subtraction, so we can cancel the last one. j is more than s, so we would go with your second option, j + s = 63 and j - s = 33. 
Can I plz have brainliest?

Answer:

The system of the equations is: j+s=63 and j-s=33

The Jackets price is j=48

The slacks prices is s=15

Step-by-step explanation:

You know for the problem that:

j+s=65

And

j= s+33; from this equation you get j-s=33

Then you replace j "j=s+33" in the first equation:

(s+33)+s=63

2s+33=63

2s=63-33

s=(63-33)/2

s=15

and you know that: j=s+33

Then, j=15+33=48

and j+s=63

48+15=63

how do you graph a function

Answers

Find the answer of the equation if it has one and mark those on the graph

The length to width ratio of a wide-screen tv is 16:9. if a tv is 30 inches long, how wide is it?

Answers

it is about 16.9 inches wide
around range its about 16.2 

Bananas cost $0.70 each, and peaches cost $0.90 each. If you want to spend exactly $21.20 at the stand and you need to buy 11 bananas, how many peaches can you buy? Write an equation in standard form modeling this situation then see how many peaches you can buy. Let be represent the number of bananas you buy and p represent the number of peaches you buy. [2 points]
Standard Form Equation: _______________________
p = ________

Answers

0.7b+0.9p = 21.2 is the equation.

You can buy 15 peaches, P = 15.

Plugging 11 in for b (equation above), you have spent $7.70 on bananas. Subtract this number from $21.20 to find that you have $13.50 left over for peaches. Divide 13.5 by 0.9 (since peaches cost 90 cents each), and your answer is 15.

With $21.20, you can buy 11 bananas and 15 peaches.

Simplify (4x3+ 13x − 7) − (6x2+ 9x + 2). −2x2+ 4x − 9 −2x2+ 22x − 5 4x3− 6x2+ 4x − 9 4x3− 6x2+ 22x − 5

Answers

(4x³+ 13x − 7) − (6x²+ 9x + 2)
Remember that if they don't have the same exponents/bases (x²,x³, etc.), then they cannot be grouped together.

This means that 4x³ and 6x² will be left alone because there are no other x² or x³ in the expression.

So first thing you do is distribute. Don't forget about that - in front of the second parentheses.
(4x³+ 13x − 7) − (6x²+ 9x + 2)
(4x³ + 13x − 7) + (−6x² - 9x - 2)
4x³ - 6x² + (13x - 7) + (-9x - 2)
4x³ - 6x² + 4x (- 7) + (- 2)
4x³ - 6x² + 4x - 9

So the answer is:
C. 4x3− 6x2+ 4x − 9

As children grow, dosages for medications gradually approach those for adults. Clark’s rule is often used to determine the correct dosage for children.

Child’s dose = Adult dose x (Weight of child in pounds)/(150 pounds)

What dosage should be administered to a child weighing 28 pounds if the adult dose of a certain drug is 80 milligrams? Round your answer to the nearest whole milligram.
How much would a child have to weigh to receive the adult dosage? Explain why your answer is reasonable.

Answers

A child weighing 28 pounds would have to take 15 milligrams of medicine in order to receive and adequate dose, if the adult dosage is 80 milligrams. For the child to receive the full adult dosage of 80 milligrams, the child would have to weigh 150 pounds.

Can you show me the steps for this equation and answerF(x)=(x-3)^2+2

Answers

Let's simplify step-by-step.
(
x

3
)
2
+
2
Distribute:
=
x
2
+

6
x
+
9
+
2
Combine Like Terms:
=
x
2
+

6
x
+
9
+
2
=
(
x
2
)
+
(

6
x
)
+
(
9
+
2
)
=
x
2
+

6
x
+
11
To solve f(x) = (x-3)^2 +2:
1. Apply the quadratic formula
2. Remeber to expand the equation
You will get x^2-6x+11, and after applying the formula, you will get imaginary numbers. I don't know if this problem is right, sorry.

The sum of negative eighteen and a number is eleven. What is the number?

Which equation could be used to solve the problem?

x - 18 = 11
18 - x = 11
-x + 18 = 11
x + 18 = 11

Answers

-18 +x = 11

rewrite as x-18 =11

x = 11+18 = 29

Answer:

x-18=11

step by step explanation

What is true about a pseudoscientific idea?

It can be replicated and verified.
It is improved with new information.
It ignores parts of the method within an investigation.
It tries to explain natural phenomena by analyzing, observing, and testing.

Answers

Pseudoscience is when the scientific method is either ignored or not applied correctly, which leads to wrong claims, data and similar issues.

so the solution is It ignores parts of the method within an investigation.


The answer is (C.) It ignores parts of the method within an investigation.

What is pseudoscience?

Pseudoscience is a proposition, a finding, or a system of explanation that is presented as science but that lacks the rigor essential to the scientific method. Pseudoscience can also be the result of research that is based on faulty premises, a flawed experimental design, or bad data.

Among the most notable developments in the history of pseudoscience in the 19th century is the rise of Spiritualism (traced in America to 1848), homeopathy (first formulated in 1796), and phrenology (developed around 1800).

Learn more about pseudoscientific here: https://brainly.com/question/604092

#SPJ2

Factor this trinomial completely. -6x3 + 26x2 + 20x   A. -2x(3x + 5)(x - 2)   B. -2x(3x + 2)(x - 5)   C. -2(3x + 2)(x - 5)   D. -2(3x + 5)(x - 2)

Answers

-6x3 + 26x2 + 20x
= -2x(3x^2 - 13x + 10)
= -2x (3x + 2)(x - 5)

answer
 B. -2x(3x + 2)(x - 5)

what is 7,925 divided by 72?

Answers

110 is the answer to your question you can type it into a calculator to make sure
110.069444444 search it up on google and it'll tell you the answer on a calculator

Ian wants to promote his band on the internet. Site a offers website hosting for $4.95 per month with a $49.99 startup fee. Site b offers website hosting for $9.95 per month with no startup fee. Write and solve an inequality to determine how many months lan could have his wedsite on site b and still keep his total cost less than site a

Answers

 I believe for twelve (12) months. 

Site A: $49.95 plus $59.40(4.95 x 12) equals $109.35 

Site B: $9.95 x 12 equals $119.40 

No, wait that's not right. 

Okay, at 8 months, site A is pretty much at $90 (forget the nickels) and site B is $80 so site B is less. 
At 7 months, site A is $85 and site B is $70 so site B is less. 
At 9 months, site A is $95 and site B is $90 so site B is less. 
At 10 months, site A $100 and site B is $100 

It's got to be around 10 months somewhere. 

Ten months would be $99.45 for site A and $99.50 for site B so B is less. 

Eleven months is $104.40 for A and $109.45 for B so now B is more.
Final answer:

Ian could have his website on site B for less than 10 months for it to be cheaper than hosting on site A.

Explanation:

To solve this question, we need to find when the costs between site a and site b equal each other. Since site a charges $4.95 per month plus a $49.99 startup fee, the cost of site a can be modelled by the equation: 4.95x + 49.99. Site b charges $9.95 per month with no startup fee, so its cost can be modelled by the equation 9.95x. We can now set up the inequality as follows:

4.95x + 49.99 > 9.95x

To solve for x, we will subtract 4.95x from both sides of the inequality, getting 49.99 > 5x. Finally, we divide each side by 5 to solve for x, resulting in x < 10. So Ian could have his website on site b for less than 10 months and still keep his total cost less than site a.

Learn more about Inequality Problem-Solving here:

https://brainly.com/question/15086723

#SPJ12

Spencer has a rectangular garden. The length and width are both odd numbers and the perimeter is 16 feet. What can be the area of his garden? Find all possible answers.

Answers

Area of rectangle = length x width

Perimeter of rectangle = 2 x (length + width)

Now we know that perimeter of rectangle = 16

2(length + width) = 16

Length + width = 16/2

Length + width = 8

Now the value of length and width is that if we add both values we have the answer 8.

So, here are some possible values of length and width and also that length and width are odd numbers;

Odd numbers are = 1,3,5,7,….

7 + 1 = 8

5 + 3 = 8

So, either length is 7 and with is 1 or length is 5 and width is 3

So area of rectangle = 7 x 1 = 7 or 5 x 3 = 15

Thus, the possible values of area of rectangle are 7 ft² and 15 ft².

 

Kyle needed to find the solution to a system of equations by graphing. According to the graph, what is the solution?

A) (2,6)
B) no solution
C) imaginary numbers
D) infinite solutions
Kyle needed to find the solution to a system of equations by graphing. According to the graph, what is the solution?

A) (2,6)
B) no solution
C) imaginary numbers
D) infinite solutions

Answers

It appears that ur lines are coincident.....they lie on top of each other....therefore, they have infinite solutions

Perimeter of the rectangular playing field is 470 yards. the length of the field is 5 yards less than triple the width, what are the dimensions of the playing field

Answers

P = 2(L + W)
P = 470
L = 3W - 5

470 = 2(3W - 5 + W)
470 = 2(4W - 5)
470 = 8W - 10
470 + 10 = 8W
480 = 8W
480/8 = W
60 = W <==== width is 60 yards

L = 3W - 5
L = 3(60) - 5
L = 180 - 5
L = 175 <==== length is 175 yards

Please help with this question thank you!

Answers

x + 5 = 2x - 1
x = 6

6 + 5 = 11 or B

The domain of a radical function is the set of all x for which the radicand is positive. True or false?

Answers

Final answer:

The domain of a radical function is the set of all x for which the radicand is positive.

Explanation:

The statement in the question is almost correct. The domain of a radical function is the set of all x for which the radicand is non-negative, not just positive.

A radical function is defined as a function that contains a radical expression, which is the square root or any higher-order root of a variable or expression. The domain of a radical function is the set of all values of x that make the radicand, which is the expression inside the radical, positive.

For example, let's consider the function f(x) = √(x+3). The radicand x+3 can only be positive for values of x greater than -3. Therefore, the domain of this function is (-3, ∞).

Learn more about Radical function here:

https://brainly.com/question/31910587

#SPJ2

A meteor crater is 3300 feet in diameter. Approximate the distance around the crater. Use 3.14 for pie.

Answers

circumference = diameter x PI

 3300 *3.14 = 10,362 feet

Answer:

Distance around the crater is 10362 feet.

Step-by-step explanation:

Given : A meteor crater is 3300 feet in diameter.

To find : Approximate the distance around the crater. Use 3.14 for pie.

Solution: We have given Diameter = 3300 feet.

Radius = [tex]\frac{Diameter}{2}[/tex].

Radius = [tex]\frac{3300}{2}[/tex].

Radius = 1650 feet.

Circumference = 2 * pi* radius.

Circumference = 2 *3.14* 1650.

Circumference = 3.14 * 3300.

Circumference = 10362 feet.

Therefore, Distance around the crater is 10362 feet.

Sue spent $40 on one pair of shoes and earned $35 baking cookies. Write an expression using positive and/or negative integers to represent this situation. Evaluate the expression and interpret what it means.

Answers

She would be $5 in debt.

.
A normal probability plot of the survival times of the guinea pigs in a medical experiment is shown below. Use this plot to describe the shape of the distribution of survival times. Then, explain carefully how this shape is seen in the normal probability plot. That is, make the connection between the normal probability plot and the shape of the data.

https://api.agilixbuzz.com/Resz/~hdAcBAAAAAQ99yZmb7l1SA.XDuW_wcASaP2dYyG_RjrDD/62689880,708,1,1,0/Assets/flvs/apstatistics_v9_gs-xml/res0019/01_16a_e.gif

Answers

the probability plot shows that the guinea pigs that survived increasingly grew quickly, most likely when they started getting all of the guinea pigs done some  came along and that is what the staggering points on the grid.

it is not d it is c i just took the test

Answers

ok so nevermind i won't help


In the triangle abc, the measure of angle a is 40 degrees, and the measure of angle b is 65 degrees. can abc be a right triangle?

Answers

All three angles of a triangle should add up to 180, no more, no less. 65+40=105, 180-105=75. So, no because the remaining angle has to be 75 degrees, 


Solve the compound inequality shown below. Show steps.
-4< 3x -1<5

Answers

Add 1 to all parts.
-4+1<3x-1+1<5+1
-3x<3x<6
-3/3<3x/3<6/3
=-1hope this helps!

The graph of a proportional relationship contains the point (8, 4).

What is the corresponding equation?

Enter your answer as a fraction in simplest form

Answers

[tex]\bf \qquad \qquad \textit{direct proportional variation}\\\\ \textit{\underline{y} varies directly with \underline{x}}\qquad \qquad y=kx\impliedby \begin{array}{llll} k=constant\ of\\ \qquad variation \end{array}\\\\ -------------------------------\\\\ (\stackrel{x}{8}~~,~~\stackrel{y}{4})\implies \begin{cases} x=8\\ y=4 \end{cases}\implies 4=k8\implies \cfrac{4}{8}=k\implies \cfrac{1}{2}=k \\\\\\ \boxed{y=\cfrac{1}{2}x}[/tex]

What is the equation for the linear model in the scatter plot obtained by choosing the two points closest to the line.
A. y= -1.5x + 6
B. y= 1.5x +6
C. y= 1.5x - 12
D y=2x+6

Answers

We can rule out choice A because the slope for choice A is negative, but the line is not moving downhill (read it from left to right)

We can also rule out choice C as the y intercept is definitely not -12. The y intercept is some positive value above 5, so 6 is probably a good guess or the correct y intercept.

This leaves choice B or choice D. They both have a y intercept of 6, so let's ignore that part. The slopes are different. For choice B we have 1.5 as the slope and choice D we have 2 as the slope.

Pick the two points closest to the line which are (4, 12) and (20,36). Start by using the graph to find the two closest points. Then use the table to confirm that you have the proper numeric values.

Slope formula:
m = (y2-y1)/(x2-x1)
m = (36-12)/(20-4)
m = 24/16
m = 1.5

So this points to choice B as the final answer

Option B is correct. The required for the linear model in the scatter plot is y = 1.5x + 6

The standard formula of expressing the equation of a line is;

y = mx + b where;

m is the slope

b is the y-intercept

Using the coordinate points (20,36) and (4, 12)

Get the slope:

[tex]Slope = \frac{12-36}{4-20}\\Slope = \frac{-24}{-16}\\Slope=\frac{3}2}=1.5[/tex]

The point where the line meets the y-axis is the t intercept. From the graph, the y-intercept is b = 6

Get the required equation:

[tex]y=mx+b\\y=1.5x + 6[/tex]

Hence the required for the linear model in the scatter plot is y = 1.5x + 6

Learn more here: https://brainly.com/question/17003809

What's the answer and how do you do it?

Answers

the vertical asymptotes for a rational expression, occur at the values of "x" that make the denominator 0, and therefore the expression undefined.

what are those anyway?  simple, just set the denominator to 0, solve for "x", and that will spit them out.

x + 4 = 0
x = -4

there, if "x" ever becomes -4, then the fraction turns to [tex]\bf \cfrac{1}{x+4}\implies \cfrac{1}{-4+4}\implies \stackrel{und efined}{\cfrac{1}{0}}[/tex]

thus, that's where the vertical asymptote is at, x = -4.
A vertical asymptote is when y gets higher and higher or lower and lower without stopping around a certain point. In a fraction one way this can happen is when there is a point where the denominator is 0, as when it gets close to 0 it gets bigger and bigger, and anything over 0 is undefined. Therefore the answer is A as 1/4-4 is 1/0 and an asymptote
Other Questions
What supreme court decision put an end to segregation? sales price is 67.50 sales tax is 5% how much is the total amount paid Water molecules are polar because the Find f. f '''(x) = cos(x), f(0) = 4, f '(0) = 2, f ''(0) = 6 ed is using the recipe shown to make fruit salad he wants to use 30 diced stawberries in his vruit sald how many bananas apples and pears should ed use in his fruit sald any measurement is accurate within 0.5 of the measurement unit. For example, if your pencil to the nearest inch, your measurement could be 0.5 inch too long or 0.5 inch too short. Write an absolute value inequality that shows the maximum and minimum actual measure of a nail measured to be 4.4 centimeters to the nearest 0.1 centimeter Which of the following countrys does the nile flow threw The great pyramid in Egypt was built around 2560 BC over the years it has lost 30 feet of height off its top and is now 451 feet tall. Find the original height of the great pyramid name three properties you could use to distinguish a proton from an electron? Who issues geotrust primary certification authority? Explain the role that religion had in different parts of the Colonies during early settlement. What type of angles are these two angles? 48, 132complementarysupplementaryparallelnone of the above Why did immigrants and farmers settle in big cities at the end of the 19th century? What happened as the number of public schools in the south began to grow? What do historians and archaeologists study in order to learn about ancient minoan culture? Paul is riding his bike 48 miles to the beach. His average rate of speed is 13.8 miles per hour. Paul stopped for a snack after 3 hours. How many miles does he have to go after his snack? Round your answer to the nearest tenth if necessary. In a dealership vehicle storage, there are 4 sections of vehicles, 10 rows per section, and 1,200 vehicles for sale. Answer #5-6. 5. If the vehicles are divided evenly among the sections, how many vehicles are in each section? 6. If the vehicles are divided evenly among the rows in each section, how many vehicles are in each row? Josh's history teacher wants josh to learn important events that occurred during the civil war. what type of knowledge would be most directly involved in this learning is shocked a mood word? btw mood means how the reader feels Over a 12 hour period from 8 pm to 8 am the tempeture fell a steady rate 8F to 16F if the temperature fell at the same rate every hour what was the temperature at 4 am Steam Workshop Downloader